You are on page 1of 8

Matthew Straughn

Math 402
401 Final Exam

Exercise 1: (a) Let g : X → R, assume f is a bounded function on X ⊂ R, let x0 be an


adherent point of X. Show that if limx→x0 g(x) = 0, then limx→x0 g(x)f (x) = 0 as well.
Proof. We know given 0 > 0 ∃δ 0 > 0 such that |g(x)| ≤ 0 for |x − x0 | < δ and also that
∃M ∈ R with |f (x)| ≤ M ∀x ∈ X as f is bounded.
 
Given  > 0 take 0 = > 0 then ∃δ > 0 with |x − x0 | < δ such that |g(x)| ≤
M M

⇒|g(x)|M ≤ 
⇒|g(x)||f (x)| ≤  (since |f (x)| ≤ M ∀x ∈ X)
⇒|g(x)f (x) − 0| ≤ 
⇒ lim g(x)f (x) = 0
x→x0

(b) Use the −δ definition of continuity to show that the linear function f (x) = ax + b is
continuous at every point x ∈ R.

Proof. Given  > 0 take δ = and consider x ∈ R such that |x − x0 | < δ
|a|

⇒|x − x0 | <
|a|
⇒|a||x − x0 | < 
⇒|ax − ax0 | < 
⇒|ax + b − ax0 − b| < 
⇒|(ax + b) − (ax0 + b)| < 
⇒|f (x) − f (x0 )| < 

Hence f is continuous at all x0 ∈ R.


Exercise 2: Let f : R → R that satisfies the multiplicative property f (x + y) = f (x)f (y)
for all x, y ∈ R. Assume f is not identically equal to zero.
1
(i) Show that f (0) = 1 and that f (−x) = f (x)
for all x ∈ R. Show that f (x) > 0 for all
x ∈ R.
First let us prove two simple Lemmas.
Lemma 1. f (0) 6= 0
Proof. Suppose f (0) = 0 then we have that f (x) = f (x + 0) = f (x)f (0) = 0 ∀x ∈ R which
is a contradiction since f is not identically equal to zero. Hence f (0) 6= 0.
Lemma 2. f (x) 6= 0 ∀x ∈ R.

1
Proof. Suppose not, that is, assume that there is some x with f (x) = 0. Then we have that
f (0) = f (x − x) = f (x)f (−x) ⇒ f (0) = 0 a contradiction from Lemma 1. Thus we have
that f (x) 6= 0 ∀x ∈ R.
Now let us prove the results in the exercise
Proof. If we take x = y = 0 then we get that f (0) = f (0)f (0) and since f (0) 6= 0 we can
divide through to get f (0) = 1.
For the next part, let us take y = −x. Then we have that:
1
f (0) = f (x − x) = f (x)f (−x) ⇒ 1 = f (x)f (−x) ⇒ f (x) = .
f (−x)
To show that f (x) > 0 ∀x ∈ R note that f (x) = f (x/2 + x/2) = f (x/2)f (x/2) = (f (x/2))2
and from Lemma 2 we have that f (x) 6= 0 ∀x ∈ R.
Therefore f (x) > 0 ∀x ∈ R.

(ii) Let a = f (1) (by (i) a > 0). Show that f (n) = an for all n ∈ N. Use (i) to show that
f (z) = az for all z ∈ Z.

Proof. (By induction on n)


Base Case: (n = 0).
a0 = 1 and from (i) f (0) = 1 this proves the base case.
Inductive Step: Assume for some n ∈ N that f (n) = an , then show true for n + 1.
f (n + 1) = f (n)f (1) = (an )a = an+1
Thus, by induction, we have that f (n) = an ∀n ∈ N.

For the second part note that if z ≥ 0 then we are done, so assume z < 0. Then by definition
we have −z = n for some n ∈ N.
1 1 1 1
Hence we have, f (z) = = = n = −z = az
f (−z) f (n) a a
Thus, we have that f (z) = az ∀z ∈ Z.

(iii) Show that f (r) = ar for all r ∈ Q.

Proof. Let r = p/q p, q ∈ Z, q 6= 0.


So, f (1) = f (1/q + 1/q + ... + 1/q ) = f (1/q)f (1/q)...f (1/q) = (f (1/q))q
| {z }
q times
⇒ (f (1/q)) = a ⇒ f (1/q) = a1/q
q

Now f (p/q) = f (1/q + 1/q + ... + 1/q ) = f (1/q)f (1/q)...f (1/q) = (f (1/q))p
| {z }
p times
⇒ f (p/q) = (f (1/q))p = (a1/q )p = ap/q = ar
Thus f (r) = ar .

(iv) Show that if f is continuous at x = 0, then f is continuous at every point in R

2
Proof. Since f is continuous at x = 0 we have that limx→0 f (x) = f (0) = 1.
From the multiplicative property of f and part (i) we get:

f (x)
lim = lim f (x)f (−x0 )
x→x0 f (x0 ) x→x0
= lim f (x − x0 )
x→x0

= lim f (x − x0 )
(x−x0 )→0

= f (0)
=1
f (x)
So we have that limx→x0 = 1 which implies that limx→x0 f (x) = f (x0 ).
f (x0 )
Therefore f is continuous at every point in R.

(v) Assume f is continuous at zero, use (iii) and (iv) to conclude that f (x) = ax for all x ∈ R
Proof. Let x ∈ R and let us take a sequence (rn )∞n=0 of rationals that approaches x. That is,
limn→∞ rn = x. From (iv) we have continuity and so we have limn→∞ f (rn ) = f (x). But from
(iii) we have that f (rn ) = arn hence we get that limn→∞ f (rn ) = ax . Thus f (x) = ax .
Exercise 3: A function f : R → R satisfies a Lipschitz condition with constant M > 0 if
for all x, y ∈ R,
|f (x) − f (y)| ≤ M |x − y|
Assume h, g : R → R each satisfy a Lipschitz condition with constant M1 and M2 respec-
tively.
(a) Show that (h + g) satisfies a Lipschitz condition with constant (M1 + M2 ).

Proof.
|(h + g)(x) − (h + g)(y)| = |h(x) + g(x) − (h(y) + g(y))|
= |h(x) − h(y) + g(x) − g(y)|
≤ |h(x) − h(y)| + |g(x) − g(y)|
≤ M1 |x − y| + M2 |x − y|
= (M1 + M2 )|x − y|

(b) Show that the composition (h ◦ g) satisfy a Lipschitz condition. With what constant?

Proof. |h(g(x)) − h(g(y))| ≤ M1 |g(x) − g(y)| ≤ M1 M2 |x − y|.


Constant is M1 M2

(c) Show that the product (hg) does not necessarily satisfy a Lipschitz condition. However,
if both functions are bounded then the product satisfies a Lipschitz condition.

3
Proof. From our Review we have that Lipschitz ⇒ Uniformly Continuous. Hence Not Uni-
formly Continuous ⇒ Not Lipschitz.
Using this take h, g = x certainly h, g satisfy Lipschitz, but hg is not uniformly continuous,
and hence does not satisfy Lipschitz.

Let h, g be bounded by A, B respectively. That is, |h(x)| ≤ A ∀x ∈ R and |g(x)| ≤ B ∀x ∈ R.

|h(x)g(x) − h(y)g(y)| = |h(x)g(x) − h(x)g(y) + h(x)g(y) − h(y)g(y)|


≤ |h(x)||(g(x) − g(y))| + |g(y)||(h(x) − h(y))|
≤ |h(x)|M2 |x − y| + |g(y)|M1 |x − y|
= (M1 |g(y)| + M2 |h(x)|)|x − y|
≤ (M1 B + M2 A)|x − y|
Hence we have that if h, g are bounded then the product hg satsfies a Lipschitz condition.
Exercise 4: (a) Assume that f : [0, ∞) → R is continuous at every point on its domain.
Show that if there exists b > 0 such that f is uniformly continuous on the set [b, ∞), then f
is uniformly continuous on [0, ∞).
Proof. Since f is continuous on [0, b] we can use Theorem 9.9.16 to conclude that f is
uniformly continuous on [0, b].
And now since f is uniformly continuous on [0, b] we know for every 0 > 0 there is some δ 0
such that |f (x) − f (y)| < 0 whenever x, y ∈ [0, b] such that |x − y| < δ 0
And since f is also uniformly continuous on [b, ∞) we know that for every 00 > 0 there is
some δ 00 such that |f (x) − f (y)| < 00 whenever x, y ∈ [b, ∞) such that |x − y| < δ 00

Now given  > 0 take 0 = /2 and 00 = /2 and then take δ = min(δ 0 , δ 00 ) so that |x − y| < δ
for x, y ∈ [0, ∞).
Now we have three cases:
(i): x, y ∈ [0, b]
(ii): x, y ∈ [b, ∞)
(iii): x < b < y

For case (i): Since f is uniformly continuous on [0, b] we have |f (x) − f (y)| < 0 < 
For case (ii): Since f is uniformly continuous on [b, ∞) we have |f (x) − f (y)| < 00 < 
For case (iii): Since x, b ∈ [0, b] we have |f (x) − f (b)| < 0 = /2 and since b, y ∈ [b, ∞) we
have |f (b) − f (y)| < 00 = /2
Thus we have:
|f (x) − f (y)| = |f (x) − f (b) + f (b) − f (y)|
≤ |f (x) − f (b)| + |f (b) − f (y)| (triangle inequality)
< /2 + /2
=

4
So |f (x) − f (y)| < 

In each case we have |f (x) − f (y)| <  thus f is uniformly continuous on [0, ∞)

(b) Prove that f (x) = x is uniformly continuous on [0, ∞).

Proof. First let us show that f is uniformly continuous on [1, ∞).


Given  > 0 take δ =  so that |x − x0 | < δ = , for x, x0 ∈ [1, ∞).

|x − x0 |  √ √
⇒√ √ ≤√ √ <  (as x + x0 > 1)
x + x0 x + x0

x − x0
⇒ √ √ <
x + x0
√ √
x − √x0 x + √x0

⇒ √ √ <
1 x + x0
√ √
⇒ | x − x0 | < 

Thus, f (x) = x is uniformly continuous on [1, ∞). Now from Proposition 9.4.11 we know
that f is continuous on [0, ∞) so with this and our proof that f is uniformly continuous on
[1, ∞) we can use part (a) to conclude that f is uniformly continuous on [0, ∞).
Exercise 5: Let {aj }j≥0 be a sequence of real numbers. Assume known that the derivative
of f (x) = ex equals f , that is, f is differentiable on R and f 0 (x) = ex .
(a) Show that f : R → (0, ∞) is invertible, and that its inverse f −1 : (0, ∞) → R is
differentiable. Find the derivative of the inverse function.

Proof. Since e > 1 we know that f (x) = ex is strictly increasing on (0, ∞) thus we can use
Proposition 9.8.3 to conclude that its inverse exists and is continuous. And since we have
that f 0 (x) = f (x) 6= 0 ∀x ∈ R we can use the Inverse Function Theorem to conclude
that f −1 has a derivative and that for f (x) = y we have:
1 1 1
(f −1 )0 (y) = = =
f 0 (x) ex y

(b) Define a function Fn : R → R for each n ∈ N by


 n
X
(n − j)xj e−jx

 if x > 0
Fn (x) = j=0

αn x + βn if x ≤ 0

Can you choose αn , βn ∈ R so that Fn is differentiable on R? Justify your answer.

5
Proof. First let us note that Fn is clearly continuous for x > 0 as we have a finite sum
of continuous functions (exponentials) products, and for x < 0 since we have a line. The
problem is at x = 0.
For Fn to be continuous at x = 0 we need:

n
X
αn × 0 + βn = lim+ (n − j)xj e−jx
x→0
j=0
" n
#
X
⇒ βn = lim+ (n)x0 e0 + (n − j)xj e−jx
x→0
j=1

⇒ βn = n + 0 = n.

Now we have that Fn is continuous whenever βn = n.

For differentiability we will need αn , the slope of our line (when x ≤ 0), to have the same
slope as our sum when x → 0+ .
That is, we need:

n
!
d X
αn = (n − j)xj e−jx
dx j=0 x=0
n
!
d −x
X
j −jx
⇒ αn = n + (n − 1)xe + (n − j)x e
dx j=2 x=0
n
!
X
⇒ αn = 0 + (n − 1)(e−x − xe−x ) + (n − j)(jxj−1 e−jx − jxj e−jx )
j=2 x=0
⇒ αn = n − 1

So if we take αn = n − 1 and βn = n then we have that Fn is differentiable on R.


Exercise 6: Let h be a differentiable function defined on the interval [0, 3], and assume that
h(0) = 1, h(1) = 2 and h(3) = 2.
(a) Show that there exists a point d ∈ [0, 3] such that h(d) = d.

Proof. Let us look at g : [1, 3] → R defined by g(x) = h(x) − x. Note that g is continuous
by continuity properties as both h and x are continuous.

g(1) = h(1) − 1 = 2 − 1 = 1
g(3) = h(3) − 3 = 2 − 3 = −1

Thus we have that g(3) ≤ 0 ≤ g(1) so take y = 0 and then by the Intermediate Value
Theorem we know that there exist d ∈ [1, 3] with g(d) = y = 0.
Hence, h(d) − d = 0 ⇒ h(d) = d

6
(b) Show that there exists a point c ∈ (0, 3) such that h0 (c) = 1/3.

Proof. This follows directly from the Mean Value Theorem which tells us that there is a
h(3) − h(0) 2−1 1
c ∈ (0, 3) such that h0 (c) = = =
3−0 3 3

(c) Show that there exists a point b ∈ (0, 3) such that h0 (b) = 1/4.

Proof. From Rolle’s Thoerem on the interval [1, 3] there is a point a ∈ [1, 3] such that
h0 (a) = 0. Using this and the point c from part (b) we have for α = 1/4 that
h0 (a) < α < h0 (c) hence by Darboux’s Theorem there is a point b ∈ (a, c) or b ∈ (c, a)
such that h0 (b) = α = 1/4. But (a, c) ⊂ (0, 3) or (c, a) ⊂ (0, 3) as a, c ∈ (0, 3).
Therefore ∃b ∈ (0, 3) with h0 (b) = 1/4.
Exercise 7: Decide whether the following statements are true or false. Justify your answers
with a couple sentences, an example, or a reference.
(a) Continuous functions take bounded closed intervals to bounded closed intervals
True from Lemma 9.6.3.
(b) The inverse image of a convergent sequence under a continuous function is necessarily a
convergent sequence.
False. Take f (x) = 1/x for x > 0, then f −1 (y) = 1/y. Now if we take yn = 1/n → 0 (as
n → ∞), then f −1 (1/n) = n → ∞.
(c) There is a continuous function on an interval that takes exactly two values.
False. Assume there is such a function, say f , and let f (a) 6= f (b) be the two distinct values
of f . Since f is continuous we can use the Intermediate Value Theorem to get any value
y with f (a) < y < f (b) or f (b) < y < f (a) contradicting that the function took on exactly
two values.
(d) If f is differentiable on [a, b], then between any two zeroes of f there must be a zero of
its derivative f 0 .
True from Rolle’s Theorem.
(e) There is a differentiable function at x0 that is not continuous at x0 .
False by Proposition 10.1.10.

Bonus: Let f : [a, b] → [a, b], assume there is c with 0 < c < 1 such that
|f (x) − f (y)| ≤ c|x − y| ∀ x, y ∈ [a, b]

(a) Show that f is uniformly continuous on [a, b].

Proof. Give  > 0 take δ = /c


Then for x, y ∈ [a, b] with |x − y| < δ we have that |x − y| < /c ⇒ c|x − y| < .
But by hypothesis, |f (x) − f (y)| ≤ c|x − y| hence |f (x) − f (y)| < .
Therefore f is uniformly continuous.

7
(b) Pick some y0 ∈ [a, b] and given yn define inductively yn+1 = f (yn ). Show that the
sequence (yn )∞
n=0 is a Cauchy sequence. Show that there is some y ∈ [a, b] such that
limn→∞ yn = y.

Proof. Let n, m > N

|f (yn−1 ) − f (ym−1 )| = |yn − ym |


= |yn − yn−1 + yn−1 − yn−2 + ... + ym+1 − ym |
≤ cn−1 |y1 − y0 | + cn−2 |y1 − y0 | + ... + cn−m |y1 − y0 |
= |y1 − y0 |(cn−m + cn−m+1 + ... + cn−1 )

But the latter part is the tail end of a geometric series with |c| < 1 and thus goes to 0. So
with N large enough we can say that |yn − ym | < . Hence (yn )∞ n=0 is Cauchy.

For the second part, we use Theorem 6.4.18 to get that (yn )∞
n=0 is a convergent sequence
and then we can use Corollary 9.1.17 to conclude that (yn )∞n=0 converges in [a, b] hence
∃y ∈ [a, b] with limn→∞ yn = y.

(c) Prove that y is a fixed point, that is, f (y) = y.

Proof. This is a result of part (b).


Since f is continuous we have limn→∞ f (yn ) = f (y)
but (f (yn )) ∼ (yn ) hence limn→∞ f (yn ) = limn→∞ (yn ) = y
Thus f (y) = y.

(d) Finally, prove that given any x ∈ [a, b], then the sequence defined inductively by:
x0 = x, xn+1 = f (xn ) converges to y as defined in part (b).

Proof. Since y0 was arbitrarily chosen from [a, b] we know that every sequence defined this
way is Cauchy and converges to a fixed point.

You might also like